Mookie betts of the boston red sox had the highest batting average for the 2018 major league baseball season. His average was 0.364. So, the likelihood of his getting a hit is 0.364 for each time he bats. Assume he has seven times at bat tonight in the red sox-yankee game. This is an example of what type of probability?

Answers

Answer 1

An example where Mookie betts of the boston red sox had the highest batting average in 2018 with probability of average batts is 0.364 for each is a binomial Probability example.

In the statistics, the concept of probability is handy in figuring out the likelihood a particular outcome would occur for an event or decision. On the basis of probability, one can figure out other related probabilities like the probability of not having that particular outcome or the probability of having only that outcome every time. There is Mookie betts of the boston red sox had the highest batting average in league baseball season.

The average probability = 0.364 for each time of bats.

That is Probability of hitting or success, p = 0.364

Number of trials, n = 7

That is Probability distribution is written as [tex]X \: \tilde \: \: Binom( n,p) [/tex].

After reading all seniror, the probability for seven seven times at bat tonight in the red sox-yankee game is an example of binomial Probability.

For more information about binomial probability, visit:

https://brainly.com/question/15278907

#SPJ4


Related Questions

Help would be much appreciated.

Answers

Answer:

A)  Rotate ΔABC 90° clockwise about the origin.

Step-by-step explanation:

From inspection of the given diagram, the coordinates of the vertices of triangle ABC are:

A = (-1, 1)B = (-1, 5)C = (-4, 2)

The coordinates of the vertices of triangle XYZ are:

X = (1, 1)Y = (5, 1)Z = (2, 4)

The mapping rule for a rotation of 90° clockwise about the origin is:

[tex]\boxed{(x, y) \rightarrow (y, -x)}[/tex]

Therefore:

A = (-1, 1)  →  X = (1, 1)B = (-1, 5)  →  Y = (5, 1)C = (-4, 2)  →  Z = (2, 4)

The mapping rule for a rotation of 90° clockwise about a point P is:

[tex]\boxed{\left([y - y_P + x_P], [x_P - x + y_P]\right)}[/tex]

So the mapping rule if the point of rotation is A (-1, 1) is:

[tex]\boxed{(y - 2 , -x)}[/tex]

Therefore:

A = (-1, 1)  →  X = (-1, 1)B = (-1, 5)  →  Y = (3, 1)C = (-4, 2)  →  Z = (0, 4)

The mapping rule for a reflection across the y -axis is:

[tex]\boxed{(x, y) \rightarrow (-x, y)}[/tex]

Therefore:

A = (-1, 1)  →  X = (1, 1)B = (-1, 5)  →  Y = (1, 5)C = (-4, 2)  →  Z = (4, 2)

The mapping rule for a reflection across the line y = x is:

[tex]\boxed{ (x, y) \rightarrow (y, x)}[/tex]

Therefore:

A = (-1, 1)  →  X = (1, -1)B = (-1, 5)  →  Y = (5, -1)C = (-4, 2)  →  Z = (2, -4)

Solution

Comparing the different transformations, we can see that the rigid motion that could be used to map triangle ABC onto triangle XYZ is:

Rotate ΔABC 90° clockwise about the origin.

Compute a confidence interval about the mean of the differences and select the correct conclusion. (A-B) A wildlife biologist wants to determine if there is a difference between two radio receivers that are used to track tagged animals with a collar. The following data represents distance (in meters) of signal from a control collar. Each burst of the signal is read by the two devices with the following data obtained: Test the biologists claim that there is a difference in the devices using a 5% level of significance. Compute a confidence interval about the mean of the differences and select the correct conclusion.

Answers

computing a confidence interval for the mean of the differences in paired data.

Compute the differences between the two measurements for each pair of data points.

Calculate the mean and standard deviation of the differences.

Compute the standard error of the mean of the differences by dividing the standard deviation of the differences by the square root of the sample size.

Determine the appropriate confidence level and degrees of freedom based on the sample size and type of test being conducted.

Use a t-distribution to find the t-value associated with the desired confidence level and degrees of freedom.

Compute the confidence interval by adding and subtracting the product of the t-value and the standard error of the mean of the differences from the sample mean of the differences.

Regarding the conclusion, if the confidence interval does not include zero, it means that there is a statistically significant difference between the two devices, and the biologist's claim is supported at the chosen level of significance. If the confidence interval includes zero, it means that there is no statistically significant difference between the two devices, and the biologist's claim is rejected at the chosen level of significance.

To learn more about confidence visit

https://brainly.com/question/29576929

#SPJ4

all correlation coefficients a) are positive. b) are negative. c) range from -1.00 to 1.00. d) use interval data.

Answers

Answer:

c) range from -1.00 to 1.00

Find F'(x): F(x) = Sx 3 t^1/3 dt

Answers

The derivative of F(x) is [tex]F'(x) = x^{(1/3)[/tex].

What is function?

A relation between a collection of inputs and outputs is known as a function. A function is, to put it simply, a relationship between inputs in which each input is connected to precisely one output.

To find the derivative of the given function F(x), we will apply the fundamental theorem of calculus and differentiate the integral with respect to x.

Let's compute F'(x):

F(x) = ∫[0 to x] [tex]t^{(1/3)} dt[/tex]

To differentiate the integral with respect to x, we'll use the Leibniz integral rule:

F'(x) = d/dx ∫[0 to x] [tex]t^{(1/3)} dt[/tex]

According to the Leibniz integral rule, we have to apply the chain rule to the upper limit of the integral.

[tex]F'(x) = x^{(1/3)} d(x)/dx - 0^{(1/3)} d(0)/dx[/tex]   [applying the chain rule to the upper limit]

Since the upper limit of the integral is x, the derivative of x with respect to x is 1, and the derivative of 0 with respect to x is 0.

[tex]F'(x) = x^{(1/3)} (1) - 0^{(1/3)} (0)[/tex]

[tex]F'(x) = x^{(1/3)[/tex]

Therefore, the derivative of F(x) is [tex]F'(x) = x^{(1/3)[/tex].

Learn more about function on:

https://brainly.com/question/7693326

#SPJ4

bank randomly selected checking account customers and found that of them also had savings accounts at the same bank. a. find the sample proportion of checking account customers also having savings accounts, . b. find the standard error of the sample proportion, . c. find a 95% confidence interval for the population proportion of checking account customers who also have savings accounts

Answers

We can say with 95% confidence that the true proportion of checking account customers who also have savings accounts in the population lies between 0.25 and 0.35.



a. To find the sample proportion of checking account customers who also have savings accounts, we need to divide the number of customers who have both types of accounts by the total number of checking account customers in the sample. Let's say the bank selected 500 checking account customers and found that 150 of them also had savings accounts. Then, the sample proportion would be:

150/500 = 0.3

So, 30% of the checking account customers in the sample also had savings accounts.

b. To find the standard error of the sample proportion, we use the formula:

SE = sqrt(p*(1-p)/n)

where p is the sample proportion (0.3 in this case), and n is the sample size (500). Plugging in the numbers, we get:

SE = sqrt(0.3*(1-0.3)/500) = 0.025

So, the standard error is 0.025.

c. To find a 95% confidence interval for the population proportion of checking account customers who also have savings accounts, we use the formula:

CI = p ± z*(SE)

where z is the z-score corresponding to a 95% confidence level (which is 1.96), and SE is the standard error we calculated in part b. Plugging in the numbers, we get:

CI = 0.3 ± 1.96*(0.025) = (0.25, 0.35)

Learn more about standard error here:

brainly.com/question/13179711

#SPJ11

question 4 consider that the following ratings exist 123456789 raterid movieid similarity with rater 30 15, 7.0 20, 30.0 assuming these are the only raters who rated movie 3285, what is the weighted average rating for the movie with id 3285?

Answers

The weighted average rating for movie 3285 can be calculated by multiplying each rater's similarity with the given rating and then dividing the sum by the total sum of similarities.

In this case, we have two raters with similarity values of 15 and 30, respectively. For the first rater with an ID of 30, the similarity value is 15 and the given rating for movie 3285 is 7.0. Therefore, the contribution of this rater to the weighted average rating is (15 x 7.0) = 105.0.



For the second rater with an ID of 20, the similarity value is 30 and no rating is given for movie 3285. Therefore, the contribution of this rater to the weighted average rating is 0. The total sum of similarities is 15 + 30 = 45.0. Thus, the weighted average rating for movie 3285 is (105.0 + 0) / 45.0 = 2.33.



So, the weighted average rating for movie 3285 is 2.33. It's important to note that this calculation assumes that these are the only two raters who rated the movie, and that their similarity values accurately reflect their taste in movies.

we'll use the following formula: Weighted Average = (Rating1 * Similarity1 + Rating2 * Similarity2) / (Similarity1 + Similarity2), Plugging in the values: Weighted Average = (15 * 7.0 + 30 * 30.0) / (7.0 + 30.0)
Weighted Average = (105 + 900) / 37
Weighted Average = 1005 / 37
Weighted Average ≈ 27.16


So, the weighted average rating for the movie with ID 3285 is approximately 27.16.

To know more about value click here

brainly.com/question/30760879

#SPJ11

consider this histogram showing the number of students in grade five who have one or more pets what is the difference in the number of students with the most and least numbers of pets?

Answers

To find the difference in the number of students with the most and least numbers of pets, we need to look at the histogram and identify the highest and lowest bars.

The histogram shows the number of students in grade five who have one or more pets, so we can assume that each bar represents a different number of pets.
Let's say the histogram shows bars for 0, 1, 2, 3, 4, and 5 pets. If the highest bar represents 12 students with 2 pets and the lowest bar represents 2 students with 0 pets, then the difference would be 10 students (12-2).
So, the answer to the question depends on the specific histogram provided. However, we can use the information in the histogram to determine the difference in the number of students with the most and least numbers of pets.

To determine the difference in the number of students with the most and least numbers of pets, please follow these steps:
1. Examine the histogram, which shows the number of students in grade five who have one or more pets.
2. Identify the column representing the most number of pets (highest bar).
3. Identify the column representing the least number of pets (lowest bar).
4. Note the number of students associated with each column (the height of the bars).
5. Calculate the difference by subtracting the number of students with the least number of pets from the number of students with the most number of pets.
Your answer: The difference in the number of students with the most and least numbers of pets in the histogram is calculated by following the steps above.

Visit here to learn more about histogram:

brainly.com/question/30354484

#SPJ11

a measure of the average value of a random variable is called a(n) group of answer choices variance. standard deviation. expected value. coefficient of variation.

Answers

The measure of the average value of a random variable is called the expected value. So, the correct answer is B).

The expected value is a measure of central tendency that represents the average value of a random variable over an infinite number of trials. It is calculated by multiplying each possible outcome by its probability of occurring, and then summing up the products.

The expected value is a useful tool in probability theory and statistics, as it provides a way to predict the long-term behavior of a random variable. For example, in a game of chance, the expected value represents the average amount of money that a player can expect to win or lose over a large number of plays.

It is also used in decision-making under uncertainty to compare different alternatives based on their expected outcomes. So, the correct option is B).

To know more about expected value:

https://brainly.com/question/29574962

#SPJ4

Find the exact length of the curve. Y = x3 3 1 4x , 1 ≤ x ≤ 2

Answers

The exact length of the curve Y = [tex]x^{3/3}[/tex] + 4x, 1 ≤ x ≤ 2 is approximately 4.526 units. The length is found using the formula for arc length integration, which involves taking the square root of the sum of squares of the first derivative of the function.

To find the exact length of the curve, we use the arc length formula

L = ∫ √[1 + (dy/dx)²] dx, where y = [tex]x^{3/4}[/tex] and 1 ≤ x ≤ 2.

Taking the derivative of y with respect to x, we get

dy/dx = 3[tex]x^{2/4}[/tex]

Substituting into the formula, we get

L = ∫ √[1 + (3[tex]x^{2/4}[/tex])²] dx

L = ∫ √[1 + 9[tex]x^{4/16}[/tex]] dx

Making the substitution u = 9[tex]x^{4/16}[/tex] + 1, du/dx = (9/4)x³, we get

L = (4/9) ∫ √(u) du

L = (4/9) * (2/3) * [tex]u^{3/2}[/tex] + C

L = (8/27) * [tex](9x^4 + 16)^{3/2}[/tex] + C

Since the curve is between x = 1 and x = 2, the exact length of the curve is

L = (8/27) * [[tex](9(2^4) + 16)^{3/2} - (9(1^4) + 16)^{3/2}[/tex]]

L = (8/27) * [[tex](160)^{3/2} - (25)^{3/2}[/tex]]

L ≈ 4.526.

Therefore, the exact length of the curve is approximately 4.526.

To know more about Length of curve:

https://brainly.com/question/31376454

#SPJ4

A certain culture of the bacterium Rhodobacter sphaeroides initially has 25 bacteria and is observed to double every 6 hours. (a) Find an exponential model n(t) = n02t/a for the number of bacteria in the culture after t hours.
Estimate the number of bacteria after 13 hours. (Round your answer to the nearest whole number.)
After how many hours will the bacteria count reach 1 million? (Round your answer to one decimal place.)

Answers

Since the culture is observed to double every 6 hours, we know that the growth rate is constant at r = ln(2)/6 per hour.

To calculate growth rates, divide the difference between the starting and ending values for the period under study by the starting value. The most frequent time intervals for growth rates are annually, quarterly, monthly, and weekly.

We can use the formula for exponential growth to model the number of bacteria in the culture after t hours:

n(t) = n0e^(rt)where n0 is the initial number of bacteria.

Substituting in the values given in the problem, we get:

n(t) = 25e^[(ln(2)/6)t]Simplifying this expression using the properties of logarithms, we can rewrite it in the form:

n(t) = 25(2)^(t/6)This is the exponential model for the number of bacteria in the culture after t hours.

To know more about  growth rate visit:

https://brainly.com/question/23879811

#SPJ4

The exponential model for population of bacteria, [tex]n(t) = n_0{2}^{\frac{t}{a} }[/tex] can be written [tex]n(t) = 25 \times {2}^{\frac{t}{6} }[/tex] for the number of bacteria in the culture after t hours. The estimate number of bacteria after 13 hours is equals to the 112. In 92 hours, the bacteria count will reach to 1 million.

We have a certain culture of the bacterium Rhodobacter.

Initial population, n₀ = 25

The population become doubles in every 6 months. The exponential model

[tex]n(t) = n_0{2}^{\frac{t}{a} }[/tex] for the number of bacteria in the culture after t hours. Now, the population become double in 6 hours, so a = 6 , then exponential equation is [tex]n(t) = 25 \times {2}^{\frac{t}{6} }[/tex].

We have to estimate the number of bacteria after 13 hours. That is t = 13 hours, [tex]n( t) = 25( 2)^{\frac{t}{6}}[/tex]

Substitute t = 13 hours

[tex] = 25( 2)^{\frac{13}{6}}[/tex]

[tex]= 25( 2)^{2.16}[/tex]

= 111.728713807 ~ 112

So, n(13) = 112

We have to determine the value of t in hours for n(t) = 1 million = 1000000, using the above equation, [tex]1000000 = 25( 2)^{\frac{t}{6}}[/tex]

[tex]40000 = ( 2)^{\frac{t}{6}}[/tex]

Taking natural logarithm both sides

=>[tex] ln( 40000) = ln(( 2)^{\frac{t}{6}})[/tex]

=> [tex]ln(40000) = \frac{t}{6} ln(2)[/tex]

=> [tex]t = \frac{6 ln( 40000)}{ ln(2)}[/tex]

= 91.7262742773 ~ 92

Hence, required value is 92 hours..

For more information about exponential model, visit:

https://brainly.com/question/29527768

#SPJ4

Find x: x2 = 20




x= ± 2√5

x=4

x= 10

x=5

Answers

The value of x is x= ± 2√5 (option a).

To solve this equation, we need to isolate x on one side of the equation. We can do this by taking the square root of both sides of the equation. However, we need to keep in mind that when we take the square root of a number, there are always two possible solutions, one positive and one negative.

So, taking the square root of both sides of x² = 20, we get:

x = ± √20

Simplifying √20, we get:

x = ± √(4 × 5)

Using the property of square roots that √(a × b) = √a × √b, we can simplify further to get:

x = ± 2√5

Therefore, the two solutions to the equation x² = 20 are x = 2√5 and x = -2√5.

However, we also need to check if any of these solutions make sense in the context of the problem. In this case, we are looking for the value of x, which is a measure of length, so we can discard the negative solution since lengths cannot be negative.

Therefore, the only valid solution is x = 2√5.

Hence the correct option is (a).

To know more about equation here

https://brainly.com/question/10413253

#SPJ4

The middle of {1, 2, 3, 4, 5} is 3. the middle of {1, 2, 3, 4} is 2 and 3. select the true statements (select all that are true) an even number of data values will always have one middle number. an odd number of data values will always have one middle value an odd number of data values will always have two middle numbers. an even number of data values will always have two middle numbers.

Answers

An even number of data values will always have two middle numbers, and an odd number of data values will always have one middle value. Therefore, the true statements are:

An even number of data values will always have two middle numbers.

An odd number of data values will always have one middle value.

What is even number?

An even number is an integer that is divisible by 2, i.e., when divided by 2, the remainder is 0. Examples of even numbers are 2, 4, 6, 8, 10, 12, etc.

The statement "an even number of data values will always have two middle numbers" is true. When there is an even number of data values, there is no single middle number because there are two values in the center.

For example, in the set {1, 2, 3, 4}, the middle numbers are 2 and 3. In general, if there are an even number of data values, the middle two values are found by taking the average of the two values in the center of the set. This is different from the case when there is an odd number of data values, where there is a single middle value.

To learn more about even number visit:

https://brainly.com/question/30758840

#SPJ4

The random variable X denotes the time taken for a computer link to be made between the terminal in an executive's office and the computer at a remote factory site. is known to have a Normal distribution, with a mean of 15 seconds and a standard deviation of 3 seconds. P(>20) has a rounded value of:

Answers

P(X > 20) has a rounded value of 0.0475.

What is mean?

By dividing the sum of the given numbers by the entire number of numbers, the mean—the average of the given numbers—is determined.

To find P(X > 20), where X is a normal random variable with mean μ = 15 seconds and standard deviation σ = 3 seconds, we need to standardize the variable and use the standard normal distribution.

Let Z be a standard normal random variable, then we can standardize X as follows:

Z = (X - μ) / σ = (20 - 15) / 3 = 1.67

Using a standard normal table or calculator, we can find the probability:

P(Z > 1.67) = 0.0475 (rounded to four decimal places)

Therefore, P(X > 20) has a rounded value of 0.0475.

Learn more about mean on:

https://brainly.com/question/14532771

#SPJ4

PLEASE HELP!!!! This question is worth 16 points and I’m stuck will give 100 points

Answers

Answer:

EF ≈ 35.4 yards

Step-by-step explanation:

to find EF use the sine ratio in the right triangle, that is

sin40° = [tex]\frac{opposite}{hypotenuse}[/tex] = [tex]\frac{EF}{DF}[/tex] = [tex]\frac{EF}{55}[/tex] ( multiply both sides by 55 )

55 × sin40° = EF , then

EF ≈ 35.4 yards ( to the nearest tenth )

there are eight households in a rural community. four of the households earn $30,000 each per year, and the other four households earn $40,000 each per year. suppose that a new resident, with an income of $2 million per year, builds a mansion in the community. after the new resident moves in, the median household income has , and the mean household income has . group of answer choices increased; increased not changed; increased increased; not changed not changed; not changed

Answers

After the new resident with an income of $2 million per year builds a mansion in the community, the median household income remains unchanged.

This is because the median household income is the middle value in a list of incomes, and the new resident's income is much higher than any of the other household incomes, so it does not affect the middle value.

However, the mean household income will increase significantly. The mean is the sum of all the incomes divided by the total number of households, and the new resident's income is much larger than any of the other households.

Therefore, when the new resident's income is added to the total income, the mean will increase significantly.

Before the new resident moved in, the total income for all eight households was $240,000 (4 households x $30,000 + 4 households x $40,000). After the new resident moves in, the total income becomes $2,240,000 ($240,000 + $2,000,000).

Dividing this by the total number of households (now nine, with the addition of the new resident) gives a new mean household income of approximately $248,888.

In conclusion, the median household income remains unchanged, while the mean household income increases significantly after the new resident with a high income moves in.

learn more about income here : brainly.in/question/15692103

Circumference of a circle

Answers

Circumference of a circle with equation [tex](x+2)^{2}+(y-3)^{2}[/tex] = 9 is 6pi.

To find the circumference of a circle with equation  [tex](x+2)^{2}+(y-3)^{2}[/tex] = 9, we first need to identify its radius, which is the square root of the constant term 9. The radius is therefore 3 units.

The formula for the circumference of a circle is C = 2πr, where C is the circumference, r is the radius, and π is a mathematical constant approximately equal to 3.14159.

Using this formula, we can calculate the circumference of the given circle as:

C = 2πr = 2π(3) = 6π

Therefore, the circumference of the circle with equation  [tex](x+2)^{2}+(y-3)^{2}[/tex] = 9 is 6π units.

It's important to note that the circumference of a circle is the distance around the edge of the circle. It is an important parameter for many applications in geometry, physics, and engineering, among others. Being able to calculate the circumference of a circle given its equation is a fundamental skill in mathematics and is essential for solving many problems in different fields.

To learn more about Circumference  here:

https://brainly.com/question/28757341

#SPJ1

a company wants to estimate how long it will take to produce 100 units of a product based on production rates in the past. which statistical method would be most effective? group of answer choices hypothesis test confidence interval regression analysis correlation analysis

Answers

A company seeking to estimate the time required to produce 100 units of a product based on past production rates should utilize regression analysis. This statistical method is the most effective among the given choices because it focuses on identifying the relationship between variables, such as production rates and time, and uses this relationship to make predictions.

Regression analysis will enable the company to develop a model that quantifies the relationship between the production rates (independent variable) and the time taken to produce units (dependent variable). By analyzing historical data, the company can establish a mathematical equation to predict future production times based on the past performance.

Hypothesis testing and confidence intervals are less suited for this purpose, as they primarily focus on determining the significance of relationships or differences between groups rather than predicting future outcomes. Similarly, correlation analysis measures the strength of a relationship between variables but does not predict future values based on past data.

In summary, regression analysis is the most effective statistical method for a company to estimate the time required to produce 100 units of a product based on past production rates. This method enables the company to create a predictive model, which can help optimize production processes and enhance overall efficiency.

To know more about hypothesis testing visit:

https://brainly.com/question/30588452

#SPJ11

On a standardized exam, the scores are normally distributed with a mean of 400 and a standard deviation of 50. Find the z-score of a person who scored 390 on the exam.

Answers

The value of the z-score of the normally distributed scores is z = -0.2

Given data ,

To find the z-score of a person who scored 390 on the exam, we can use the formula for z-score:

z = (X - μ) / σ

where:

X = the score of the person = 390

μ = the mean of the distribution = 400

σ = the standard deviation of the distribution = 50

On simplifying , we get

z = (390 - 400) / 50

z = -10 / 50

z = -0.2

Hence , the z-score of a person who scored 390 on the exam is -0.2

To learn more about z-score click :

https://brainly.com/question/15016913

#SPJ1

Determine if the columns of the matrix form a linearly independent set. Justify your answer.

Answers

To determine if the columns of a matrix form a linearly independent set, we need to check if the only solution to the equation Ax = 0 is the trivial solution, where x is a vector of coefficients and 0 is a vector of zeros. If the only solution is the trivial solution, then the columns of the matrix are linearly independent. If there is a non-trivial solution, then the columns of the matrix are linearly dependent.

Without knowing the matrix in question, I cannot provide a specific answer to this question. However, the process for determining linear independence is as described above.

I need this in 3 minutes

Answers

Answer: 142

Step-by-step explanation:

Suppose we roll a fair six-sided die and sum the values obtained on each roll, stopping once our sum exceeds 354. Approximate the probability that at least 94 rolls are needed to get this sum

Answers

The approximate probability that at least 94 rolls are needed to get a sum greater than 354 is 0.852.

X be the number of rolls needed to obtain a sum greater than 354. We are interested in finding P(X ≥ 94).

We can use the fact that the sum of two fair six-sided dice is uniformly distributed between 2 and 12. Thus, the sum of n rolls of a fair six-sided die is uniformly distributed between n and 6n.

Let Yn be the sum of the first n rolls of the die. Then Yn is uniformly distributed between n and 6n, and we have:

P(Yn > 354) = P(Yn - n > 354 - n) = P((Yn - n)/5 > (354 - n)/5)

Now, (Yn - n)/5 is uniformly distributed between 1 and 6, and (354 - n)/5 is between 1 and 70. So we have:

P(Yn > 354) = P((Yn - n)/5 > (354 - n)/5) = P(U > (354 - n)/5)

where U is a uniform random variable on [1,6].

We want to find P(X ≥ 94) = P(Y94 ≤ 354) = 1 - P(Y94 > 354) = 1 - P(U > (354 - 94)/470) = 1 - (70/471) = 0.852.

Therefore, the approximate probability that at least 94 rolls are needed to get a sum greater than 354 is 0.852.

Probability is the likelihood or chance of an event. Occurring for example, the probability of flipping a coin and it being heads is ½, because there is 1 way of getting a head and the total number of possible outcomes is 2 (a head or tail). We write P(heads) = ½ .

To know more about probability visit:

https://brainly.com/question/11234923

#SPJ4

which of the following is true about bayes' theorem? it can be used only for cases where conditional probabilities are unknown. it is useful for determining optimal decisions without requiring knowledge of probabilities of the states of nature. it enables the use of sample information to revise prior probabilities. it cannot be used to calculate posterior probabilities.

Answers

Bayes' Theorem is a mathematical theorem that enables the revision of prior probabilities based on new information or evidence.

This theorem is widely used in statistics, machine learning, and other fields that deal with uncertainty and probabilistic reasoning. Contrary to the first option mentioned in the question,

Bayes' Theorem can be used when conditional probabilities are known, and it enables the calculation of posterior probabilities, which is the probability of a hypothesis or event given the available evidence.

Therefore, the third option is correct; Bayes' Theorem enables the use of sample information to revise prior probabilities. This theorem is highly valuable because it allows the integration of new data or knowledge into the decision-making process,

which can lead to more accurate predictions and better-informed decisions. In summary, Bayes' Theorem is a powerful tool that requires knowledge of probabilities and enables the calculation of posterior probabilities based on new evidence or information.

By combining prior probabilities with likelihoods (based on new data), we can calculate posterior probabilities, which represent our updated knowledge.

This process is crucial in making informed decisions in various fields, such as data science, finance, and medical diagnosis.

To know more about theorem click here

brainly.com/question/30242664

#SPJ11

Outside temperature over a day can be modeled as a sinusoidal function. Suppose you know the temperature varies between 33 and 57 degrees during the day and the average daily temperature first occurs at 10 AM. How many hours after midnight, to two decimal places, does the temperature first reach 42 degrees?

Answers

Tthe temperature first reaches 42 degrees 7.67 hours after midnight, or approximately at 7:40 AM.

The temperature variation over a day can be represented as a sinusoidal function in the form of y = A sin(Bx - C) + D, where A is the amplitude, B is the frequency, C is the phase shift, and D is the vertical shift.

In this case, the midline of the temperature function is (33 + 57)/2 = 45 degrees. Therefore, D = 45.

The amplitude of the function is (57 - 33)/2 = 12 degrees. Therefore, A = 12.

Since the average temperature first occurs at 10 AM, which is 10 hours after midnight, the phase shift can be determined as C = (10/24) * 2π.

To find the frequency B of the function, we need to use the fact that the temperature function repeats every 24 hours. Therefore, B = 2π/24 = π/12.

Putting all the values in the equation y = 12 sin(π/12(x - 5/3)) + 45, we need to solve for x when y = 42.

42 = 12 sin(π/12(x - 5/3)) + 45

-3 = 12 sin(π/12(x - 5/3))

-1/4 = sin(π/12(x - 5/3))

π/2 = π/12(x - 5/3)

x - 5/3 = 6

x = 23/3

Therefore, the temperature first reaches 42 degrees 7.67 hours after midnight, or approximately at 7:40 AM.

Learn more about Temperature:

https://brainly.com/question/25677592

#SPJ4

suppose x > 0 and (xi) is an in nite sequence converging to x. show that for some n 2 n; that xi > 0; i > n . g

Answers

To prove that for some n > 0, xi > 0 for all i > n, we can use the fact that (xi) is a sequence that converges to x.

This means that given any ε > 0, there exists an N such that |xi - x| < ε for all i > N.  we can use the fact that (xi) is a sequence that converges to x.

Let's choose ε = x/2. Since x > 0, ε > 0 as well. Then there exists an N such that |xi - x| < ε = x/2 for all i > N. Rearranging this inequality, we get:

x - xi < x/2

xi > x/2

Since xi > x/2 and x > 0, we have shown that for some N > 0, xi > 0 for all i > N.

Learn more about Sequences

https://brainly.com/question/6561461

#SPJ4

g if f is uniformly continuous on a~ r, and fl(x)l > k > 0 for all x e a, show that 1/f is uniformly continuous on a.

Answers

It is shown that for any ε > 0, there exists a δ > 0 such that |x - y| < δ implies |1/f(x) - 1/f(y)| < ε/(2kM) for all x, y in a. This proves that 1/f is uniformly continuous on a.

What is uniformly continuous?

Uniform continuity is a property of a function in which for any given value ε > 0, there exists a corresponding value δ > 0 such that for all pairs of points in the function's domain whose distance is less than δ, the difference in the function's values at those points is less than ε. In other words, a function is uniformly continuous if its rate of change does not vary significantly over its entire domain, and small changes in its input result in correspondingly small changes in its output.

To show that 1/f is uniformly continuous on a, we need to prove that for any ε > 0, there exists a δ > 0 such that |x - y| < δ implies |1/f(x) - 1/f(y)| < ε for all x, y in a.

Given that f is uniformly continuous on a, we know that for any ε > 0, there exists a δ > 0 such that |x - y| < δ implies |f(x) - f(y)| < ε/k for all x, y in a.

We also know that |f(x)| > k for all x in a.

Using these facts, we can begin by manipulating the expression |1/f(x) - 1/f(y)|:

|1/f(x) - 1/f(y)| = |(f(y) - f(x))/(f(x)f(y))|

Since |f(y) - f(x)| < ε/k, we can substitute this into the above expression:

|1/f(x) - 1/f(y)| < |(ε/k)/(f(x)f(y))|

Now, we need to find a way to relate f(x)f(y) to |x - y|.

Since f is uniformly continuous, we know that for any ε > 0, there exists a δ > 0 such that |x - y| < δ implies |f(x) - f(y)| < ε/k for all x, y in a.

This implies that |f(x)f(y)| < k(f(x) + f(y)) < 2kM, where M is the supremum of |f(x)| over a.

Thus, we have:

|1/f(x) - 1/f(y)| < ε/(2kM)

Therefore, we have shown that for any ε > 0, there exists a δ > 0 such that |x - y| < δ implies |1/f(x) - 1/f(y)| < ε/(2kM) for all x, y in a. This proves that 1/f is uniformly continuous on a.

To know more about Uniformly continuous visit:

https://brainly.com/question/31693678

#SPJ4

without considering the sizes of the wedges, how do the three pie charts differ in which functions they include?

Answers

The three pie charts differ in the functions they include based on the distribution of the wedges.

Even without considering the sizes of the wedges, we can see that the first pie chart includes three functions while the second includes four and the third includes five.

In the first pie chart, the wedges are distributed evenly, representing three different functions. On the other hand, in the second pie chart, the wedges are not evenly distributed, with one wedge taking up more space than the others. This indicates that one function is more prominent in the second pie chart. Finally, in the third pie chart, the wedges are distributed in a way that shows one function taking up almost half of the chart.

Therefore, even without considering the sizes of the wedges, we can tell that the three pie charts differ in which functions they include based on the distribution of the wedges.

The first pie chart represents an even distribution of functions, the second pie chart has one function being more prominent, and the third pie chart has one function being the most significant.

Learn more about pie charts here:

brainly.com/question/24207368

#SPJ11

A truck driver earns $40 per hour, and an engineer earns $80 per hour. If the truck driver made 1,440 this week

Answers

The total cost for both the truck driver and the engineer this week is $4,320.

To solve this problem, we first need to determine how many hours the truck driver worked this week. We can do this by dividing the weekly earnings of the truck driver by their hourly rate of $40:

1440 / 40 = 36 hours

Therefore, the truck driver worked for 36 hours this week.

Now, we need to determine the total earnings of the engineer for the same number of hours. We know that the engineer earns $80 per hour, so for 36 hours of work, they will earn:

80 x 36 = $2,880

Therefore, the total cost for both the truck driver and the engineer this week will be the sum of their earnings:

1440 (truck driver) + 2880 (engineer) = $4,320

To know more about total here

https://brainly.com/question/14286201

#SPJ4

Complete Question:

A truck driver earns $40 per hour, and an engineer earns $80 per hour. If the truck driver made 1,440 this week, then the total cost does he made is?

In any one-minute interval, the number of requests for a popular Web page is a Poisson random variable with expected value 180 requests.a A Web server has a capacity of C requests per minute. If the number of requests in a one-minute interval is greater than C, the server is overloaded. Use the central limit theorem to estimate the smallest value of C for which the probability of overload is less than 0.055. Note that your answer must be an integer. Also, since this is a discrete random variable, don't forget to use "continuity correction". C= b Now assume that the server's capacity in any one-second interval is âC/60â, where âxâ is the largest integer â¤x. (This is called the floor function.) For the value of C derived in part (a), what is the probability of overload in a one-second interval? This time, don't approximate via the CLT, but compute the probability exactly.

Answers

Poisson distribution of number of requests for a popular Web page,

a) Web server has a capacity of C requests per minute is equals to the 206.

b) The probability of overload in a one-second interval is approximately equal to 1.

Let x denotes the number of requests for a popular web page. Now, X = number of requests per minute ~ Poisson (180)

Now, by central limit theorem the distribution of x can be approximated by Normal diet with mean = 180 and variance 180 and we denote the approximated variable by Y, that is [tex]Y \: \tilde \: \: N(180, 180)[/tex].

If number of requests in a one minute interval is greater than C, then probability of overload is less than 0.055, that is P[ X > C] < 0.055

P[ X > C] ~ P[ Y > C + 0.5] ( by continuity )

so, P[ Y > C + 0.5] < 0.055

[tex]P[ \frac{ Y - 180}{ \sqrt{180}} > \frac{C + 0.5 - 180}{ \sqrt{180} }] < 0.055[/tex]

According to normal distribution, [tex] P[ \frac{ Y - 180}{180} ] = Z ≃N(1,0)[/tex]

Therefore, [tex]P[ Z > \frac{C + 0.5 - 180}{ \sqrt{180} }] < 0.055[/tex]

=> [tex][\frac{C + 0.5 - 180}{ \sqrt{180} }] < Z_{0.055}[/tex]

= 0.478069 ~ 0.4781.

=> [tex]C - 199.5 < 0.4781 × \sqrt{ 180} [/tex]

=> C = 199.5 + 0.4781 × 13.4164

=> C = 205.91 ~ 206.

b) Now, we have to determine the probability of overload in a one-second interval, using the value of C obtained in part(a), so, C = 206 so, [ C/60] = 3

Probability of overload, P = P( X> 3)

= 1 - P( X≤ 3)

[tex]= 1 - \sum_{x = 0}^{3} e^{-180} \frac{ 180^x}{x!} [/tex]

= 1

Hence, required probability is 1.

For more information about Possion distribution, visit:

https://brainly.com/question/30890396

#SPJ4

Solve for a. Round to the nearest tenth, if necessary.
B
56
x
22°
D

Answers

Answer:

[tex]x \approx 60.4[/tex]

Step-by-step explanation:

We can solve for x using the trigonometric ratio cosine:

[tex]\cos(\theta) = \dfrac{\text{adjacent}}{\text{hypotenuse}}[/tex]

↓ plugging in the given values

[tex]\cos(22\°) = \dfrac{56}{x}[/tex]

↓ taking the reciprocal of (flipping) both sides

[tex]\dfrac{1}{\cos(22\°)} = \dfrac{x}{56}[/tex]

↓ multiplying both sides by 56

[tex]\dfrac{56}{\cos(22\°)} = x[/tex]

↓ plugging into a calculator

[tex]\boxed{x \approx 60.4}[/tex]

A result is called statistically significant when ever

Answers

A result is called statistically significant whenever it is unlikely to have occurred by chance alone, meaning that there is strong evidence to support the presence of a true effect or relationship.

This is often determined by a p-value less than a predetermined threshold, commonly set at 0.05, which indicates a less than 5% probability that the result is due to chance.

A result is called statistically significant whenever it is unlikely to have occurred by chance alone. This is typically determined by conducting a hypothesis test and calculating a p-value, which represents the probability of obtaining the observed result or a more extreme result if the null hypothesis (i.e. no difference between groups or no relationship between variables) is true.

If the p-value is below a predetermined significance level (often set at 0.05), then the result is considered statistically significant, meaning there is evidence to reject the null hypothesis and support the alternative hypothesis (i.e. there is a difference between groups or a relationship between variables).

Visit here to learn more about probability  : https://brainly.com/question/30034780
#SPJ11

Other Questions
In "no witchcraft for sale," how does gideon treat teddy when he is a child?. when solutions of kcl and pb(no3)2 are mixed, a precipitate forms. which of the following is the balanced equation for the double replacement reaction that occurs? which is the most accurate and precise way to rewrite this sentence?the recycling commission suggests in its posters that viewers can help create a safer, more sustainable world by properly sorting their cans, bottles, and paper goods for pickup.the recycling commission shows in its posters that recycling will make the world a better, healthier place, and asks that viewers take the same point of view. When a business incorporates, it must file its ______ with the state in which it incorporates. The boxplot shown below results from the heights (cm) of males listed in a data set. What do the numbers in that boxplot tell us? 153 174.7 194 what happens during the performing stage of team development: group of answer choices all of these task completions demonstration of hidden agendas lay out of operating rules you recommend splitting the software qa function between genovia and baltonia. how should the work be divided between them? select an option from the choices below and click submit. baltonia should perform most or all of the content qa and genovia should perform most or all of the functional qa. baltonia should receive one half of the qa projects and genovia should receive the other half. baltonia should perform most or all of the functional qa and genovia should perform most or all of the content qa. antibodies bind to specific antigens because of the three-dimensional shape of the variable regions. true or false how many moles of gas(air) are in the lungs of the average adult with the lung capacity of 3.8 L. Assume the person is at 1.00 atm pressure and has a normal body temperature of 37 degrees celsius. to find the portfolio's average return, we have to multiply each stock's average returns by... group of answer choices just the dollars invested in each stock. the dollar amount invested in the stock divided by the total dollars invested. one divided by the total number of stocks the stocks' standard deviations the southern division of barstol company makes and sells a single product, which is a part used in manufacturing trucks. the annual production capacity is 40,000 units and the variable cost of each unit is $38. presently the southern division sells 37,000 units per year to outside customers at $50 per unit. the northern division of barstol company would like to buy 20,000 units a year from southern to use in its production. there would be no savings in variable costs from transferring the units internally rather than selling them externally. the lowest acceptable transfer price from the standpoint of the southern division should be closest to: Which type of skeletal muscle is associated with stamina and endurance?. a random variable x has the following probability distribution. x f(x) 0 0.27 1 0.35 2 0.05 3 0.25 4 0.08 (a) determine the expected value of x. (b) determine the variance. Explain the differences between the British, French, and the Spanish in their approaches to settling the new world. Also explain why the British succeeded and the Spanish and French failed. As Blood Alcohol Concentration (BAC) increases, the area of the brain that controls muscular movements begins to slow down - the brain takes longer to process information and react. Muscular reactions become slow; steering and braking movements become uncoordinated.T/F """To lose one parent, Mr Worthing, may be regarded as a misfortune; to lose both looks like carelessness.""""""to be born ... in a handbag ... seems to me to display a contempt for the the ordinary decencies of family life"""What does it mean? What type of drug, generally speaking, is Verapamil? I.e., what is it's mechanism of action? Which inequality describes the elevations of the starfish in the tide pool One form of energy that exists in every system but is difficult to quantify is heat. Think about how we formulated our spring resonance model. Did we account for the heat energy in the medium? why do we need to?. GE operates a company trading site where it posts purchasing needs, invites bids, negotiates terms, and places orders. This is an example of a(n) ________.A) company buying siteB) intranet linkC) search engineD) trading exchangeE) reverse auction